Đến nội dung

Hình ảnh

VMF's Marathon Bất Đẳng Thức Olympic

marathon aops vmf

  • Please log in to reply
Chủ đề này có 162 trả lời

#41
hknguyen1904

hknguyen1904

    Lính mới

  • Pre-Member
  • 9 Bài viết

Một lời giải mới bài 14 ạ :|

$\frac{a+b}{ab+1}+\frac{b+c}{bc+1}+\frac{c+a}{ca+1}\geq \frac{9}{5}$

Ta có: $\frac{a+b}{ab+1}+\frac{b+c}{bc+1}+\frac{c+a}{ca+1} =\frac{(a+b)^{2}}{(a+b)(ab+1)}+\frac{(b+c)^{2}}{(b+c)(bc+1)}+\frac{(c+a)^{2}}{(c+a)(ca+1)}$ $\geq \frac{(2(a+b+c))^{2}}{ab(a+b)+bc(b+c)+ca(c+a)+2(a+b+c)} (CS) =\frac{4}{2+ab(1-c)+bc(1-a)+ca(1-b)}$$= \frac{4}{2+ab+bc+ca-3abc}$

ĐPCM <=> $\frac{4}{2+ab+bc+ca-3abc}\geq \frac{9}{5} <=>2+27abc\geq 9(ab+bc+ca)$ (1)

Áp dụng bdt Schur ta có:

$abc\geq (a+b-c)(b+c-a)(c+a-b)=(1-2a)(1-2b)(1-2c)=1-2(a+b+c)+4(ab+bc+ca)-8abc <=>9abc>=4(ab+bc+ca)-1 <=>27abc>=12(ab+bc+ca)-3$

Thay $27abc\geq 12(ab+bc+ca)-3$ vào (1) ta có: $ab+bc+ca \leq \frac{1}{3}$

Bđt đúng do a + b + c = 1. Ta có đpcm. Đẳng thức xảy ra <=> $a = b = c = \frac{1}{3}$

Ngược dâu rồi bạn!



#42
Gachdptrai12

Gachdptrai12

    Thượng sĩ

  • Điều hành viên THCS
  • 280 Bài viết

Bài 15. Cho $a,b,c>0$ sao cho $ab+bc+ca=1$. Chứng minh
$$(a^2+ab+b^2)(b^2+bc+c^2)(c^2+ca+a^2)\ge\frac{2}{3}(\frac{a}{b+c}+\frac{b}{c+a}+\frac{c}{a+b}).$$

Lời giải bài 15

Ta sẽ đồng bậc hóa bất đẳng thức

$\Leftrightarrow(a^{2}+ab+b^{2})(b^{2}+bc+c^{2})(c^{2}+ca+a^{2})\ge\frac{2}{3}(\frac{a}{b+c}+\frac{b}{c+a}+\frac{c}{a+b})(ab+bc+ca)^{3}$
Bây giờ ta sẽ chứng minh
$2(\sum \frac{a}{b+c})\leq \frac{2(a^{2}+b^{2}+c^{2})+ab+bc+ca}{ab+bc+ca}$

$\Leftrightarrow 2(\sum \frac{a}{b+c})-3\geq \frac{2(a^{2}+b^{2}+c^{2}-ab-bc-ca)}{ab+bc+ca}$

$\Leftrightarrow \sum \frac{(a-b)^{2}}{(a+c)(b+c)}\leq \frac{\sum(a-b)^{2}}{ab+bc+ca}$

$\Leftrightarrow \sum \frac{c^{2}(a-b)^{2}}{(ab+bc+ca)(a+c)(b+c)}\geq 0$

Vậy bổ đề được chứng minh ta sẽ chứng minh 1 bất đẳng thức mạnh hơn
$(a^{2}+ab+b^{2})(b^{2}+bc+c^{2})(c^{2}+ca+a^{2})\geq \frac{1}{3}(2\sum a^{2}+\sum ab)(ab+bc+ca)^{2}$
$\Leftrightarrow 3(a^{2}+ab+b^{2})(b^{2}+ca+c^{2})(c^{2}+ca+a^{2})-(2\sum a^{2}+\sum ab)(ab+bc+ca)^{2}\geq 0$

$\Leftrightarrow \sum a^{2}b^{2}(a^{2}+b^{2})-abc[\sum a^{3}+\sum ab(a+b)+3abc]+2\sum a^{3}b^{3}\geq 0$
Ta chú ý các bất đẳng thức sau
$2\sum a^{3}b^{3}=\sum a^{3}(b^{3}+c^{3})\geq \sum a^{3}bc(b+c)=\sum abc^{3}(a+b)$
$\sum a^{2}b^{2}(a^{2}+b^{2})=\sum a^{4}(b^{2}+c^{2})\geq 2abc(a^{3}+b^{3}+c^{3})\geq abc(\sum a^{3}+3abc)$
Từ đây ta có điều cần chứng minh

 

Bài toán 16. (Võ Quốc Bá Cẩn,Trần Quang Hùng) Cho $a,b,c$ là các số thực không âm thỏa $a+b+c=1$. Chứng minh

$\dfrac{1}{\sqrt{(a^2+ab+b^2)(a^2+ac+c^2)}}+\dfrac{1}{\sqrt{(b^2+bc+c^2)(b^2+ba+a^2)}}+\dfrac{1}{\sqrt{(c^2+ca+a^2)(c^2+cb+b^2)}}\geq 4+\dfrac{8}{\sqrt{3}}$


Bài viết đã được chỉnh sửa nội dung bởi hoanglong2k: 29-05-2016 - 13:25


#43
trungvmfcsp

trungvmfcsp

    Binh nhì

  • Thành viên mới
  • 13 Bài viết

Ngược dâu rồi bạn!

Ở nhể mình quên, xin lỗi  :))


Bài viết đã được chỉnh sửa nội dung bởi trungvmfcsp: 28-05-2016 - 21:25


#44
Nguyenhuyen_AG

Nguyenhuyen_AG

    Trung úy

  • Thành viên nổi bật 2016
  • 945 Bài viết
Bài 14:(Micheal Rosenberg)

cho $a,b,c$ dương thỏa a+b+c=1 chứng minh

$\frac{a+b}{ab+1}+\frac{b+c}{bc+1}+\frac{c+a}{ca+1}\geq \frac{9}{5}$

 

Lời giải bài 14. Viết bất đẳng thức lại dưới dạng

\[(a+b+c)\sum \frac{a+b}{(a+b+c)^2+ab}\geqslant \frac{9}{5}.\]

Chuyển về pqr như sau

\[p^4(p^2-4q)+2p(3p^2+5q)r-9r^2 \geqslant 0,\]

hay là

\[2p(11p^2+6q)r+(p^2-4q)(3p^4-q^2)+\left [ p^2q^2-4q^3+2p(9q-2p^2)r-27r^2 \right ] \geqslant 0.\]

\[p^2q^2-4q^3+2p(9q-2p^2)r-27r^2 = (a-b)^2(b-c)^2(c-a)^2 \geqslant 0,\]

nên ta chỉ cần chứng minh

\[2p(11p^2+6q)r+(p^2-4q)(3p^4-q^2) \geqslant 0. \quad (1)\]

Do $p^2 \geqslant 3q > 0$ vì thế

  • Nếu $p^2 \geqslant 4q$ thì $(1)$ đúng.
  • Nếu $p^2 \leqslant 4q$ theo bất đẳng thức Schur bậc bốn ta có $r \geqslant \frac{(4q-p^2)(p^2-q)}{6p}.$ Ta cần chỉ ra \[2p(11p^2+6q) \cdot \frac{(4q-p^2)(p^2-q)}{6p}+(p^2-4q)(3p^4-q^2) \geqslant 0,\] thu gọn thành \[\frac{1}{3}(4q-p^2)(p^2-3q)(2p^2+q) \geqslant 0.\] Hiển nhiên đúng.

Chứng minh hoàn tất.


Bài viết đã được chỉnh sửa nội dung bởi Nguyenhuyen_AG: 28-05-2016 - 23:01

Nguyen Van Huyen
Ho Chi Minh City University Of Transport

#45
Nguyenhuyen_AG

Nguyenhuyen_AG

    Trung úy

  • Thành viên nổi bật 2016
  • 945 Bài viết

$$(a^{2}+ab+b^{2})(b^{2}+bc+c^{2})(c^{2}+ca+a^{2})\geq \frac{1}{3}(2\sum a^{2}+\sum ab)(ab+bc+ca)^{2}$$

 

Bất đẳng thức này đúng mọi số thực $a,b,c$ bất kỳ vì nó tương đương với

\[\frac{1}{3}\left [ a^2b^2+b^2c^2+c^2a^2-abc(a+b+c) \right ](a+b+c)^2 \geqslant 0.\]

 

 \begin{array}{| l | l |} \hline \text{HDTterence2k} & 1\\ \hline \text{hoanglong2k} & 3\\ \hline \text{Gachdptrai12} & 7\\ \hline \text{Nguyenhuyen_AG} & 5\\ \hline \text{fatcat12345} & 4\\ \hline \text{lenhatsinh3} & 1\\ \hline \text{tuanyeubeo2000} & 1\\ \hline \text{Ngockhanh99k48} & 1 \\ \hline \text{Dinh de Tai} & 1\\ \hline \end{array} 


Bài viết đã được chỉnh sửa nội dung bởi hoanglong2k: 29-05-2016 - 09:47

Nguyen Van Huyen
Ho Chi Minh City University Of Transport

#46
hoanglong2k

hoanglong2k

    Trung úy

  • Điều hành viên THCS
  • 965 Bài viết

 Lời giải bài 16. Giả sử $c=\min \{a,b,c\}$. Trước tiên ta chứng minh $\dfrac{1}{\sqrt{b^2+bc+c^2}}+\dfrac{1}{\sqrt{a^2+ac+c^2}}\geq \dfrac{2\sqrt{2}}{\sqrt{2ab+ac+bc+2c^2}}$

Tương đương với 

$\begin{align*}2\left(\dfrac{1}{b^2+bc+c^2}+\dfrac{1}{a^2+ac+c^2}-\dfrac{4}{2ab+ac+bc+2c^2}\right)&\geq \left(\dfrac{1}{\sqrt{b^2+bc+c^2}}-\dfrac{1}{\sqrt{a^2+ac+c^2}}\right)^2\\ \Leftrightarrow \dfrac{2(a-b)^2(2ab+bc+ca-c^2)}{(a^2+ac+c^2)(b^2+bc+c^2)(2ab+ac+bc+2c^2)}&\geq \dfrac{(a-b)^2(a+b+c)^2}{(a^2+ac+c^2)(b^2+bc+c^2)\left(\sqrt{a^2+ac+c^2}+\sqrt{b^2+bc+c^2}\right)^2}\\ \Leftrightarrow \dfrac{2(2ab+bc+ca-c^2)}{2ab+ac+bc+2c^2}&\geq \dfrac{(a+b+c)^2}{\left(\sqrt{a^2+ac+c^2}+\sqrt{b^2+bc+c^2}\right)^2} \end{align*}$

 Hiển nhiên đúng do $\dfrac{2(2ab+bc+ca-c^2)}{2ab+ac+bc+2c^2}\geq 1\geq \dfrac{(a+b+c)^2}{\left(\sqrt{a^2+ac+c^2}+\sqrt{b^2+bc+c^2}\right)^2}$

 Do đó $\text{VT}\geq \dfrac{2\sqrt{2}}{\sqrt{(a^2+ab+b^2)(2ab+ac+bc+2c^2)}}+\dfrac{1}{\sqrt{(c^2+ca+a^2)(c^2+cb+b^2)}}$

 Đặt $t=ab$ và xét $f(t)=\dfrac{2\sqrt{2}}{\sqrt{\left[(1-c)^2-t\right](2t+c^2+c)}}$

 Có $f'(t)=\dfrac{\sqrt{2}(4t+5c-c^2-2)}{(c^2-2c-t+1)(c^2+c+2t)\sqrt{(c^2-2c-t+1)(c^2+c+2t)}}$

 Mà $4t+5c-c^2-2\leq (a+b)^2+5c-c^2-2=(1-c)^2+5c-c^2-2=3c-1\leq 0$ nên $f(t)$ nghịch biến

 Suy ra $f(t)\geq f\left [\dfrac{(a+b)^2}{4}\right]=\dfrac{8}{\sqrt{3(a+b)^2\left[4c^2+2c(a+b)+(a+b)^2\right]}}=\dfrac{8}{\sqrt{3(1-c)^2(1+3c^2)}}$

 Mặt khác lại có $\left[4c^2+2c(a+b)+(a+b)^2\right]^2-16(a^2+ac+c^2)(b^2+bc+c^2)$

                          $=(a-b)^2(a^2+6ab+4ac+b^2+4bc-4c^2)\geq 0$

 Cho nên $\dfrac{1}{\sqrt{(c^2+ca+a^2)(c^2+cb+b^2)}}\geq \dfrac{4}{4c^2+2c(a+b)+(a+b)^2}=\dfrac{4}{3c^2+1}$

 Vậy nên $\text{VT}\geq \dfrac{8}{\sqrt{3(1-c)^2(1+3c^2)}}+\dfrac{4}{3c^2+1}$

 Xét $g(c)=\dfrac{8}{\sqrt{3(1-c)^2(1+3c^2)}}+\dfrac{4}{3c^2+1}$ trên $\left[0,\dfrac{1}{3}\right]$ ta được $\min g(c)=g(0)=\dfrac{8}{\sqrt{3}}+4$

 Vậy ta có điều cần chứng minh. Dấu "=" xảy ra khi $a=b=\dfrac{1}{2},c=0$ cùng các hoán vị

 

 Bài toán 17. (Sưu tầm) Cho các số thực không âm $x,y,z$ thỏa mãn $x+y+z=32$. Tìm giá trị lớn nhất của

$P=x^3y+y^3z+z^3x$


Bài viết đã được chỉnh sửa nội dung bởi hoanglong2k: 29-05-2016 - 21:43


#47
Nguyenhuyen_AG

Nguyenhuyen_AG

    Trung úy

  • Thành viên nổi bật 2016
  • 945 Bài viết
Bài toán 16. (Võ Quốc Bá Cẩn,Trần Quang Hùng) Cho $a,b,c$ là các số thực không âm thỏa $a+b+c=1$. Chứng minh $$\dfrac{1}{\sqrt{(a^2+ab+b^2)(a^2+ac+c^2)}}+\dfrac{1}{\sqrt{(b^2+bc+c^2)(b^2+ba+a^2)}}+\dfrac{1}{\sqrt{(c^2+ca+a^2)(c^2+cb+b^2)}}\geq 4+\dfrac{8}{\sqrt{3}}$$

 

Lời giải bài 16. Đặt $k=\frac{2}{\sqrt{3}}$ áp dụng bất đẳng thức Holder, ta có

\[\left [\sum\frac{1}{\sqrt{(a^2+ab+b^2)(b^2+bc+c^2)}}  \right ]^2 \geqslant \frac{(k+1)^3(a+b+c)^3}{\displaystyle \sum (a^2+ab+b^2)(b^2+bc+c^2)\left [ (k+1)a+b+c \right ]^3}.\]

Chú ý rằng $\left (4+\frac{8}{\sqrt{3}}  \right )^2 = 16(k+1)^2$ nên ta quy bài toán về chứng minh

\[(k+1)(a+b+c)^7 \geqslant \sum (a^2+ab+b^2)(b^2+bc+c^2)\left [ (k+1)a+b+c \right ]^3.\]

Chuyển về pqr như sau

\[Ar + p(p^2-4q)B \geqslant 0. \quad (1)\]

Trong đó

\[\left\{\begin{aligned}A & = \frac{16}{3}(102-83k)p^4+192(7k-8)p^2q+128(6-5k)q^2, \\ B &= \frac{729-615k}{81423}\left [ 9047p^4+4(6876k+251)qp^2+64(1-297k)q^2 \right ]\end{aligned}\right.\]

Ta sẽ chứng minh $A,\,B \geqslant 0.$ Thật vậy, đặt $p^2 = (t+3)q$ với $t \geqslant 0,$ ta được

\[\left\{\begin{aligned}A & = \frac{16\left [ (102-83k)t^2+(324-246k)t+198-111k \right ]p^4}{3(t+3)^2} \geqslant 0, \\ B& = \frac{(243-205k)\left [ 9047t^2+(27504k+55286)t+63504k+84499 \right ]p^4}{27141(t+3)^2} \geqslant 0.\end{aligned}\right.\]

Như vậy

  • Nếu $p^2 \geqslant 4q$ thì $(1)$ đúng.
  • Nếu $p^2 \leqslant 4q$ áp dụng bất đẳng thức Schur bậc bốn $r \geqslant \frac{(4q-p^2)(p^2-q)}{p}$ ta đưa bài toán về chứng minh \[A \cdot \frac{(4q-p^2)(p^2-q)}{6p} + p(p^2-4q)B \geqslant 0,\] hay là \[\frac{A(p^2-q)}{6p} - pB \geqslant 0. \quad (2)\] Cũng đặt $p^2 = (t+3)q$ với $t \geqslant 0,$ ta thấy $(2)$ trở thành \[\frac{\left (261-98\sqrt{3} \right )\left [  13103t^3+(23184\sqrt{3}+37411)t^2+(57888\sqrt{3}+39453)t+19152\sqrt{3}+22929\right ]p^5}{353781(t+3)^3} \geqslant 0.\] Hiển nhiên đúng

Vậy ta có điều phải chứng minh.

 

P/s. Anh nghĩ lời giải của Long không đúng, bài này chắc có kiểu dồn biến theo kiểu thừa trừ của anh Cẩn. Còn điều kiện của bài 17dương hay không âm nhỉ ?


Bài viết đã được chỉnh sửa nội dung bởi Nguyenhuyen_AG: 29-05-2016 - 21:27

Nguyen Van Huyen
Ho Chi Minh City University Of Transport

#48
fatcat12345

fatcat12345

    Binh nhất

  • Banned
  • 46 Bài viết

Lời giải bài 17.

Đặt $f(x,y,z)=x^3y+y^3z+z^3x$

Vì $f$ là hàm hoán vị theo $x,y,z$ nên ta xét 2 trường hợp

Trường hợp 1: $x\leq y\leq z$

Ta xét 

$d_1=f(x,y,z)-f(x,y+z,0)=z(y^3+z^2x-x^3)$

$d_2=f(x,y,z)-f(0,y,z+x)=x(z^3+x^2y-y^3)$

$d_3=f(x,y,z)-f(x+y,0,z)=y(x^3+y^2z-z^3)$

Vì $(y^3+z^2x-x^3)+(x^3+y^2z-z^3)+(z^3+x^2y-y^3)=z^2x+y^2z+x^2y\geq 0$

nên ta lại xét 2 trường hợp

   Trường hợp 1.1. $d_1,d_2,d_3\geq 0$

   Suy ra $z^3\leq x^3+y^2z$

   Xét $f(x,y,z)-f(0,y+\frac{x}{2},z+\frac{x}{2})$

         $=\frac{1}{16}x[(16z^3-16x^3-16y^2z)+(15x^3-12xy^2-3xyz)+(10x^2y-8y^2z-2zx^2)-8y^3-9xyz]\leq 0$

   Đến đây ta chỉ cần xét trường hợp 1 biến bằng 0 sẽ xét ở cuối bài.

    Trường hợp 1.2. Tồn tại $d_i\leq 0$. Trở về trường hợp xét 1 biến bằng 0.

Trường hợp 2: $x\geq y\geq z$

Ta xét $f(x,y,z)-f(x+\frac{z}{2},y+\frac{z}{2},0)$

           $=\frac{1}{16}[(10z^2x-10x^2y)+(16y^3-8x^3-8x^2y)-6x^2y-12zx^2-12xyz-2yz^2-z^3]\leq 0$

 Tất cả trường hợp đều dẫn tới việc xét 1 biến bằng 0

 Nên ta chỉ cần tìm Max của $x^3y$ với $x+y=32$

 Việc này rất dễ dàng

 $x^3y=27\left ( \frac{x}{3} \right )^3y\leq \frac{27}{256}(x+y)^4=110592$

 Vậy $Max\; f(x)= 110592$ khi và chỉ khi $(x,y,z)=(24,8,0)$ và các hoán vị vòng quanh.

 

Bài toán 18. (AoPS) Cho $a,b,c$ là các số không âm thỏa $ab+bc+ca>0$. Chứng minh

$\dfrac{1}{a^2+b^2}+\dfrac{1}{b^2+c^2}+\dfrac{1}{c^2+a^2}\geq \dfrac 4{a^2+b^2+c^2}+\dfrac{1}{2(ab+bc+ca)}$


Bài viết đã được chỉnh sửa nội dung bởi hoanglong2k: 29-05-2016 - 22:11


#49
Gachdptrai12

Gachdptrai12

    Thượng sĩ

  • Điều hành viên THCS
  • 280 Bài viết

 Bài toán 17. (Nguyễn Quốc Anh) Cho các số thực dương $x,y,z$ thỏa mãn $x+y+z=32$. Tìm giá trị lớn nhất của

$P=x^3y+y^3z+z^3x$

 Lời giải : Giả sử $x=\max \{x,y,z\}$

Ta có

$a^{3}b+b^{3}c+c^{3}a\leq a^{3}b+a^{2}bc+a^{2}c^{2}\leq a^{3}b+a^{2}bc+\frac{a^{2}c(a+c)}{2}=a^{2}(a+c)\left(b+\frac{c}{2}\right)$

Áp dụng AM-GM ta có

$a^{2}(a+c)\left(b+\frac{c}{2}\right)=27\frac{a^{2}(a+c)}{27}\left(b+\frac{c}{2}\right)\leq \frac{27}{256}\left(\frac{a}{3}+\frac{a}{3}+\frac{a+c}{3}+b+\frac{c}{2}\right)^{4}= \frac{27}{256}\left(a+b+\frac{5c}{6}\right)^{4}\leq \frac{27}{256}(a+b+c)^{4}=\frac{32^{4}.27}{256}$


Bài viết đã được chỉnh sửa nội dung bởi hoanglong2k: 29-05-2016 - 22:05


#50
Gachdptrai12

Gachdptrai12

    Thượng sĩ

  • Điều hành viên THCS
  • 280 Bài viết

P/s. Anh nghĩ lời giải của Long không đúng, bài này chắc có kiểu dồn biến theo kiểu thừa trừ của anh Cẩn. Còn điều kiện của bài 17dương hay không âm nhỉ ?

lời giải của Long em nghĩ nó đúng mà với lại đó là dồn biến thừa trừ luôn mà anh Huyện



#51
Nguyenhuyen_AG

Nguyenhuyen_AG

    Trung úy

  • Thành viên nổi bật 2016
  • 945 Bài viết

lời giải của Long em nghĩ nó đúng mà với lại đó là dồn biến thừa trừ luôn mà anh Huyện

 

À anh nhìn nhầm.


Nguyen Van Huyen
Ho Chi Minh City University Of Transport

#52
hoanglong2k

hoanglong2k

    Trung úy

  • Điều hành viên THCS
  • 965 Bài viết

Lời giải bài 18. Ta viết bất đẳng thức thành $\dfrac{\sum a^4+3\sum a^2b^2}{\prod (a^2+b^2)}\geq \dfrac{4}{a^2+b^2+c^2}+\dfrac{1}{2(ab+bc+ca)}$

 Đặt $p=a+b+c=1,q=ab+bc+ca,r=abc$ thì ta cần chứng minh $\dfrac{1-4q+5q^2-2r}{q^2-2q^3-2r+4qr-r^2}\geq \dfrac{1+6q}{2q(1-2q)}$

 Hay là $r^2(1+6q)+r(1+4q)(1-2q)+q(2-q)(1-4q)(1-2q)\geq 0$

 Nếu $q\leq \dfrac{1}{4}$ thì $r^2(1+6q)+r(1+4q)(1-2q)+q(2-q)(1-4q)(1-2q)\geq q(2-q)(1-4q)(1-2q)\geq 0$

 Nếu $\dfrac{1}{3}\geq q\geq \dfrac{1}{4}$, áp dụng bất đẳng thức Schur bậc 4 ta có $r\geq \dfrac{(4q-1)(1-q)}{6}$

 Do đó ta chỉ cần chứng minh $\left[\dfrac{(4q-1)(1-q)}{6}\right]^2(1+6q)+\dfrac{(4q-1)(1-q)(1+4q)(1-2q)}{6}+q(2-q)(1-4q)(1-2q)\geq 0$

 Tương đương với $(3q-1)(4q-1)(8q^3-6q^2+27q-11)\geq 0$, tức là ta chỉ cần chứng minh $f(q)=8q^3-6q^2+27q-11\leq 0$ với $\dfrac{1}{3}\geq q\geq \dfrac{1}{4}$

 Mặt khác, $f'(q)=24q^2-12q+27>0$ nên $f(q)\leq f\left(\dfrac{1}{3}\right)=\dfrac{-64}{27}<0$

 Do đó ta có điều cần chứng minh. Dấu "=" xảy ra khi $a=b=c$ hoặc $a=b,c=0$ cùng các hoán vị

 

 Một lời giải khác cho bài 17.

 Một lời giải khác nữa :)

 

  Bài toán 19. (AoPS) Cho $x,y,z\geq 0$ thỏa mãn $x+y+z=32$. Chứng minh rằng

\[x^3y+y^3z+z^3x+\dfrac{473xyz}{8}\leq 110592\]

 

 \begin{array}{| l | l |} \hline \text{HDTterence2k} & 1\\ \hline \text{hoanglong2k} & 5\\ \hline \text{Gachdptrai12} & 8\\ \hline \text{Nguyenhuyen_AG} & 6\\ \hline \text{fatcat12345} & 5\\ \hline \text{lenhatsinh3} & 1\\ \hline \text{tuanyeubeo2000} & 1\\ \hline \text{Ngockhanh99k48} & 1 \\ \hline \text{Dinh de Tai} & 1\\ \hline \end{array} 


Bài viết đã được chỉnh sửa nội dung bởi hoanglong2k: 30-05-2016 - 10:37


#53
Nguyenhuyen_AG

Nguyenhuyen_AG

    Trung úy

  • Thành viên nổi bật 2016
  • 945 Bài viết

 Bài toán 17. (Sưu tầm) Cho các số thực không âm $x,y,z$ thỏa mãn $x+y+z=32$. Tìm giá trị lớn nhất của

$$P=x^3y+y^3z+z^3x$$

 

Lời giải bài 17. Bản chất của bài toán này chính là bất đẳng thức

\[x^3y+y^3z+z^3x \leqslant \frac{27}{256}(x+y+z)^4.\]

Ta sẽ chứng minh kết quả mạnh hơn của Michael Rozenberg

\[x^3y+y^3z+z^3x+\frac{473}{256}xyz(x+y+z) \leqslant \frac{27}{256}(x+y+z)^4,\]

hay là

\[256(x^3y+y^3z+z^3x)+473xyz(x+y+z) \leqslant 27(x+y+z)^4. \quad (1)\]

Giả sử $z = \min\{x,y,z\}$ và đặt $x=a+z,\;y=b+z$ bất đẳng thức (1) trở thành

\[217(a^2-ab+b^2)z^2+(68a^3-269a^2b+499ab^2+68b^3)z+(27a^2+14ab+3b^2)(a-3b)^2 \geqslant 0.\]

Chú ý rằng

\[68a^3-269a^2b+499ab^2+68b^3 = 60a(a^2-5ab+8b^2)+8a^3+31a^2b+19ab^2+68b^3 \geqslant 0.\]

Nên ta có điều phải chứng minh.


Bài viết đã được chỉnh sửa nội dung bởi Nguyenhuyen_AG: 29-05-2016 - 23:59

Nguyen Van Huyen
Ho Chi Minh City University Of Transport

#54
Nguyenhuyen_AG

Nguyenhuyen_AG

    Trung úy

  • Thành viên nổi bật 2016
  • 945 Bài viết
Bài toán 18. (AoPS) Cho $a,b,c$ là các số không âm thỏa $ab+bc+ca>0$. Chứng minh

$\dfrac{1}{a^2+b^2}+\dfrac{1}{b^2+c^2}+\dfrac{1}{c^2+a^2}\geq \dfrac 4{a^2+b^2+c^2}+\dfrac{1}{2(ab+bc+ca)}$

 

Lời giải bài 18. Bất đẳng thức cần chứng minh tương đương với

\[\dfrac{a^2}{b^2+c^2}+\dfrac{b^2}{c^2+a^2}+\dfrac{c^2}{a^2+b^2} \geqslant 1+\dfrac{a^2+b^2+c^2}{2(ab+bc+ca)},\]

hay là

\[\dfrac{a^2}{b^2+c^2}+\dfrac{b^2}{c^2+a^2}+\dfrac{c^2}{a^2+b^2} \geqslant \dfrac{(a+b+c)^2}{2(ab+bc+ca)}.\]

Hiển nhiên đúng do

\[\sum \frac{a^2}{b^2+c^2} \geqslant \sum \frac{a}{b+c}\geqslant \dfrac{(a+b+c)^2}{2(ab+bc+ca)}.\] Chứng minh hoàn tất.

 


Bài viết đã được chỉnh sửa nội dung bởi Nguyenhuyen_AG: 30-05-2016 - 00:13

Nguyen Van Huyen
Ho Chi Minh City University Of Transport

#55
quykhtn-qa1

quykhtn-qa1

    Binh nhất

  • Thành viên
  • 23 Bài viết
Xin được đính chính bài 19 tác giả không phải như bạn hoanglong2k viết mà là một bài toán cũ trên AoPS. Bản chất của bài toán là  bất đẳng thức sau:
Cho các số thực không âm $x,y,z$. Chứng minh rằng
$$x^3y+y^3z+z^3x+\frac{473}{256}xyz(x+y+z) \leqslant \frac{27}{256}(x+y+z)^4.$$
 
Lời giải trên AoPS (giống với lời giải của bạn Nguyenhuyen_AG) là sử dụng phương pháp BW, dùng tính toán khá nhiều và có lẽ không đơn giản nếu không dùng máy tính. Về bài toán này, chúng ta có thể giải như sau:
Trường hợp $x+y+z=0$, tức là $x=y=z=0$ thì bất đẳng thức hiển nhiên đúng. Xét trường hợp $x+y+z>0$, không mất tổng quát giả sử $x+y+z=1$ và $y$ nằm giữa $x,z$. Ta có $$y^3z+z^3x=z^3y+xy^2z+z(y^2-z^2)(y-x)\leq z^3y+xy^2z.$$ Do đó, ta chỉ cần chứng minh $$x^3y+z^3y+xy^2z+\dfrac{473xyz}{256} \leq \dfrac{27}{256},$$ $$ 256\big[y(x+z)^3-3xyz(x+z)+xy^2z\big]+473xyz \leq 27,$$ $$ 256y(1-y)^3+xz(1024y^2-295y) \leq 27.$$ $\bullet$ Nếu $y\leq \dfrac{295}{1024}$, ta có
$$ VT\leq 256y(1-y)^3=256\cdot 27\cdot y\left(\frac{1-y}{3}\right)^3\leq  256\cdot 27\left(\dfrac{y+3\cdot \frac{1-y}{3}}{4}\right)^4=27.$$
$\bullet$ Nếu $y>\dfrac{295}{1024}$, ta có $xz\leq y(x+z-y)=y(1-2y)$. Do đó, ta chỉ cần chứng minh $$ 256y(1-y)^3+y(1-2y)(1024y^2-295y) \leq 27,$$ $$ 2304y^4-2382y^3+1063y^2-256y+27\geq 0,$$ $$ (3y-1)^2(256y^2-94y+27)\geq 0.$$  Bất đẳng thức cuối hiển nhiên đúng. Đẳng thức xảy ra khi và chỉ khi $x=y=z$ hoặc $x=3y,\ z=0$, hoặc các hoán vị tương ứng. 

 



#56
quykhtn-qa1

quykhtn-qa1

    Binh nhất

  • Thành viên
  • 23 Bài viết

Lời giải bài 16. Đặt $k=\frac{2}{\sqrt{3}}$ áp dụng bất đẳng thức Holder, ta có

\[\left [\sum\frac{1}{\sqrt{(a^2+ab+b^2)(b^2+bc+c^2)}}  \right ]^2 \geqslant \frac{(k+1)^3(a+b+c)^3}{\displaystyle \sum (a^2+ab+b^2)(b^2+bc+c^2)\left [ (k+1)a+b+c \right ]^3}.\]

Chú ý rằng $\left (4+\frac{8}{\sqrt{3}}  \right )^2 = 16(k+1)^2$ nên ta quy bài toán về chứng minh

\[(k+1)(a+b+c)^7 \geqslant \sum (a^2+ab+b^2)(b^2+bc+c^2)\left [ (k+1)a+b+c \right ]^3.\]

Chuyển về pqr như sau

\[Ar + p(p^2-4q)B \geqslant 0. \quad (1)\]

Trong đó

\[\left\{\begin{aligned}A & = \frac{16}{3}(102-83k)p^4+192(7k-8)p^2q+128(6-5k)q^2, \\ B &= \frac{729-615k}{81423}\left [ 9047p^4+4(6876k+251)qp^2+64(1-297k)q^2 \right ]\end{aligned}\right.\]

 

Trong lời giải này, bước chuyển pqr cần tính toán khá nhiều, chứ không phải một bước ra luôn như dùng phần mềm máy tính. Mình nghĩ trong topic này chỉ nên đăng các bất đẳng thức và lời giải có thể tính toán bằng tay thôi.
 

Xin được đề xuất bài toán sau:

Bài toán 20. (AoPS)  Cho các số thực không âm $a,b,c$ thỏa mãn  $a+b+c=3.$ Chứng minh rằng 

\[ a^3b^2+b^3c^2+c^3a^2+7(ab+bc+ca) \leq 24.\]


Bài viết đã được chỉnh sửa nội dung bởi hoanglong2k: 30-05-2016 - 10:35


#57
hoanglong2k

hoanglong2k

    Trung úy

  • Điều hành viên THCS
  • 965 Bài viết

Trong lời giải này, bước chuyển pqr cần tính toán khá nhiều, chứ không phải một bước ra luôn như dùng phần mềm máy tính. Mình nghĩ trong topic này chỉ nên đăng các bất đẳng thức và lời giải có thể tính toán bằng tay thôi.
 

Xin được đề xuất bài toán sau:

Bài toán 20. (AoPS)  Cho các số thực không âm $a,b,c$ thỏa mãn  $a+b+c=3.$ Chứng minh rằng 

\[ a^3b^2+b^3c^2+c^3a^2+7(ab+bc+ca) \leq 24.\]

 Một lời giải "xấu xí" của em cho bài toán này

 Lời giải bài 20.

 Tương tự như anh Quý, viết lại bất đẳng thức về dạng đồng bậc là 

$$81(a^3b^2+b^3c^2+c^3a^2)+21(ab+bc+ca)(a+b+c)^3\leq 8(a+b+c)^5$$

 Không mất tính tổng quát, giả sử $b$ nằm giữa $a$ và $c$, khi đó ta sẽ có $c(bc-c^2)(b^2-a^2)=c^2(b+a)(b-c)(b-a)\leq 0$

 Do đó mà $b^3c^2+c^3a^2\leq b^2c^3+a^2bc^2$

 Ta cần chứng minh $81b\left[b(a^3+c^3)+a^2c^2\right]+21\left[b(c+a)+ca\right](a+b+c)^3\leq 8(a+b+c)^5$

 Tương đương $81b\left[b(a+c)^3-3abc(a+c)+a^2c^2\right]+21\left[b(c+a)+ca\right](a+b+c)^3\leq 8(a+b+c)^5$

 Chuẩn hóa $a+b+c=1$ và đặt $t=ac$ thì do $(b-a)(b-c)\leq 0\Rightarrow t\leq b(a+c-b)=b(1-2b)$, từ đây cũng suy ra $b\leq \dfrac{1}{2}$

 Ta cần chứng minh

$$81b\left[b(1-b)^3-3bt(1-b)+t^2\right]+21\left[b(1-b)+t\right]\leq 8$$

$$\Leftrightarrow 81b(-b^4+3b^3-3b^2+b-3bt+3b^2t+t^2)+21(-b^2+b+t)\leq 8$$

$$\Leftrightarrow 81bt^2+t(243b^3-243b^2+21)\leq 81b^5-243b^4+243b^3-60b^2-21b+8$$

 Mà $t\leq b(1-2b)$ nên ta chỉ cần chứng minh

$$81tb(1-2b)+t(243b^3-243b^2+21)\leq 81b^5-243b^4+243b^3-60b^2-21b+8$$

$$\Leftrightarrow 3t(3b+1)(9b^2-21b+7)\leq 81b^5-243b^4+243b^3-60b^2-21b+8$$

 Bây giờ, nếu $9b^2-21b+7\leq 0$ tức là $b\geq \dfrac{7-\sqrt{21}}{6}>0,4$

 Khi này ta chỉ cần chứng minh $81b^5-243b^4+243b^3-60b^2-21b+8\geq 0$ với $b\in \left[\dfrac{7-\sqrt{21}}{6};\dfrac{1}{2}\right]$ là đủ (thực ra nó đúng với mọi $b\geq 0$ thì phải)

 Đoạn này mình chứng minh không được đẹp lắm, phải chia nhỏ ra để làm

 Đặt $f(b)=81b^5-243b^4+243b^3-60b^2-21b+8$ với $b\in \left[\dfrac{7-\sqrt{21}}{6};\dfrac{1}{2}\right]$

 Có $f'(b)=3(135b^4-324b^3+243b^2-40b-7),\ f''(b)=6(270b^3-486b^2+243b-20)>0$ nên $f'(b)$ đồng biến.

  Trường hợp $0,4<b\leq 0,443$ thì $f'(b)\leq f'(0,443)<0$ nên $f(b)\geq f(0,443)>0$

  Trường hợp $0,5\geq b\geq 0,4431$ thì $f'(b)\geq f'(0,4431)>0$ nên $f(b)\geq f(0,4431)>0$

  Còn mà $0,443<b<0,4431$ thì $81b^5-243b^4+243b^3-60b^2-21b+8$

                                                      $\geq 81.0,443^5-243.0,4431^4+243.0,443^3-60.0,4431^2-21.0,4431+8>0$

 Do đó $f(b)>0$ và trường hợp này đúng

 Nếu $9b^2-21b+7\geq 0$, do $t\leq b(1-2b)$ nên ta chỉ cần chứng minh

$$3b(1-2b)(3b+1)(9b^2-21b+7)\leq 81b^5-243b^4+243b^3-60b^2-21b+8$$

$$\Leftrightarrow (3b-1)^2(27b^3-54b^2+6b+8)\geq 0$$

 Bất đẳng thức trên đúng với mọi $0\leq b\leq \dfrac{1}{2}$ nên ta có điều cần chứng minh

 Dấu "=" xảy ra khi và chỉ khi $a=b=c=1$


Bài viết đã được chỉnh sửa nội dung bởi hoanglong2k: 31-05-2016 - 00:14


#58
quykhtn-qa1

quykhtn-qa1

    Binh nhất

  • Thành viên
  • 23 Bài viết

Bài toán 20. (AoPS)  Cho các số thực không âm $a,b,c$ thỏa mãn  $a+b+c=3.$ Chứng minh rằng 

\[ a^3b^2+b^3c^2+c^3a^2+7(ab+bc+ca) \leq 24.\]

Lời giải của em cơ bản là chuẩn rồi, hoanglong2k. Bài toán cũng có thể chứng minh đơn giản hơn như sau:
Không mất tổng quát, giả sử $b$ nằm giữa $a$ và $c,$ tức là $(b-a)(b-c) \leq 0.$ Khi đó \[ a^3b^2+b^3c^2+c^3a^2=c^2(b^2-a^2)(b-c)+b^2(a^3+c^3)+b(ac)^2 \leq b^2(a^3+c^3)+b(ac)^2.\]
Ta chỉ cần chứng minh
\[ b^2(a^3+c^3)+b(ac)^2+7(ab+bc+ca) \leq 24,\] hay
\[ f(x)=bx^2-\left[3b^2(3-b)+7\right]x+b^2(3-b)^3+7b(3-b) \leq 24,\]
với $x=ac,\ \ 0\leq ac\leq b(3-2b).$ Để ý rằng $f(x)$ là một tam thức bậc hai với hệ số $b \geq 0$. Do đó ta chỉ cần chứng minh $ f(0) \leq 24 $ và $f\left[ b(3-2b)\right] \leq 24.$ Thật vậy
\[ \begin{aligned} f(0)& = b^2(3-b)^3+7b(3-b)\\ & =24-\dfrac{(3b-4)^2}{9}\left(b^3-\dfrac{19b^2}{3}+\dfrac{25b}{3}+\dfrac{364}{27}\right)-\dfrac{33b+8}{243} <24, \end{aligned} \] \[ \begin{aligned}  f\left[ b(3-2b)\right] \ & =b^3(3-2b)^2-\left[3b^2(3-b)+7\right]b(3-2b)+b^2(3-b)^3+7b(3-b) \\ & =24-3(b-1)^2(b^3-6b^2+2b+8) \leq 24.\end{aligned}\]
Phép chứng minh hoàn tất. Đẳng thức xảy ra khi và chỉ khi $a=b=c=1.$


#59
quykhtn-qa1

quykhtn-qa1

    Binh nhất

  • Thành viên
  • 23 Bài viết

Bài toán 21. (AoPS) Cho các số thực không âm $a,b,c$ thỏa mãn  $a+b+c=3.$ Chứng minh rằng 

\[\sqrt{4a^4+5abc}+\sqrt{4b^4+5abc}+\sqrt{4c^4+5abc} \geq 9.\]


Bài viết đã được chỉnh sửa nội dung bởi hoanglong2k: 31-05-2016 - 15:38


#60
Nguyenhuyen_AG

Nguyenhuyen_AG

    Trung úy

  • Thành viên nổi bật 2016
  • 945 Bài viết

Bài toán 21. (AoPS)  Cho các số thực không âm $a,b,c$ thỏa mãn  $a+b+c=3.$ Chứng minh rằng 

\[\sqrt{4a^4+5abc}+\sqrt{4b^4+5abc}+\sqrt{4c^4+5abc} \geq 9.\]

 

Lời giải bài 21. Vì \[\sqrt{4a^4+5abc}-2a^2 = \frac{5abc}{2a^2+\sqrt{4a^4+5abc}},\]

và $9 = \sum a^2 + 2\sum bc,$ nên bất đẳng thức cần chứng minh tương đương với

\[5abc \sum \frac{1}{2a^2+\sqrt{4a^4+5abc}} \geqslant 2(ab+bc+ca) -(a^2+b^2+c^2).\]

Áp dụng bất đẳng thức Schur bậc ba

\[2(ab+bc+ca) -(a^2+b^2+c^2) \leqslant \frac{9abc}{a+b+c} = 3abc,\]

ta quy bài toán về chứng minh

\[\sum \frac{1}{2a^2+\sqrt{4a^4+5abc}} \geqslant \frac{3}{5}.\]

Theo bất đẳng thức AM-GM thì $bc \leqslant \frac{(b+c)^2}{4} = \frac{(3-a)^2}{4},$ và

\[\sqrt{a(4a^3+5bc)} \leqslant \frac{1}{6}\left [ 9a+(4a^3+5bc) \right ] \leqslant \frac{1}{24}(16a^3+5a^2+6a+45).\]

nên ta chỉ cần chỉ ra

\[\sum \frac{24}{16a^3+53a^2+6a+45} \geqslant \frac{3}{5}.\]

Bất đẳng thức này được suy ta từ bất đẳng thức sau và hai bất đẳng thức nữa tương tự

\[\frac{24}{16a^3+53a^2+6a+45} \geqslant \frac{1}{5} - \frac{4}{15}(a-1).\]

Nhưng điều này hiển nhiên vì nó tương đương với

\[\frac{(64a^2+228a+45)(a-1)^2}{15(16a^3+53a^2+6a+45)} \geqslant 0.\]

Bài toán được chứng minh.


Bài viết đã được chỉnh sửa nội dung bởi Nguyenhuyen_AG: 31-05-2016 - 14:53

Nguyen Van Huyen
Ho Chi Minh City University Of Transport





Được gắn nhãn với một hoặc nhiều trong số những từ khóa sau: marathon, aops, vmf

2 người đang xem chủ đề

0 thành viên, 2 khách, 0 thành viên ẩn danh